Đến nội dung

CaptainCuong nội dung

Có 201 mục bởi CaptainCuong (Tìm giới hạn từ 24-04-2020)



Sắp theo                Sắp xếp  

#704469 Đề thi học sinh giỏi lớp 9 Tp HCM

Đã gửi bởi CaptainCuong on 29-03-2018 - 17:41 trong Tài liệu - Đề thi

Bài 5.2 Áp dụng định lí brahamagupta suy ra M,N lần lượt là trung điểm AD, BC. Tới đây quen thuộc rồi




#700366 ĐỀ KIỂM TRA LỚP CHUYÊN LẦN 3 - THPT CHUYÊN LÊ HỒNG PHONG, THÀNH PHỐ HỒ CHÍ MINH

Đã gửi bởi CaptainCuong on 15-01-2018 - 23:52 trong Thi HSG cấp Tỉnh, Thành phố. Olympic 30-4. Đề thi và kiểm tra đội tuyển các cấp.

 

Câu 4. Cho $A$ là tập hợp gồm $n$ phần tử là các số nguyên dương phân biệt ($n>1$) sao cho khi bớt đi một phần tử bất kỳ của $A$ thì tập hợp còn lại có thể chia được thành 2 tập con (có giao khác rỗng) sao cho tổng các phần tử ở mỗi tập con bằng nhau. Chứng minh các phần tử của $A$ cùng tính chẵn lẻ và $n\geq 7$.

Câu 5. Cho tam giác nhọn không cân $ABC$, có đường trung tuyến $AM$ và đường cao $AH$ ($M,H\in BC$). Các điểm $Q$ và $P$ lần lượt thuộc các tia $AB$ và $AC$ sao cho $QM\perp AC$ và $PM\perp AB$. Đường tròn ($PMQ$) cắt cạnh $BC$ lần thứ hai tại điểm $X$. Chứng minh rằng $BH=CX$.

 

https://artofproblem...126020p5204821 

All Russia MO 2015




#698746 Chứng minh $T$ thuộc trung tuyến của tam giác $ABC$.

Đã gửi bởi CaptainCuong on 22-12-2017 - 08:44 trong Hình học

Cho $\Delta ABC(AB<AC)$ nội tiếp đường tròn $(O).$ Tiếp tuyến tại $B$ và $C$ cắt nhau tại $P. AO, AC$ cắt $(BOC)$ lần lượt tại $L$ và $Q.T$ là giao điểm của $PQ$ và $BL.$ Chứng minh $T$ thuộc trung tuyến của tam giác $ABC.$




#697808 TOPIC Luyện tập về ứng dụng của tỉ số kép và hàng điểm điều hòa

Đã gửi bởi CaptainCuong on 04-12-2017 - 23:37 trong Hình học

Bài 3. (9) VMO 2010 

 

Cho tam giác $ABC$ nội tiếp $(O)$ cố định $B,C$  và $A$ di chuyển trên $(O)$. Gọi phân giác trong và ngoài của tam giác lần lượt là $AD$ và $AE$ với $D,E$ thuộc $BC$. $M$ là trung điểm của $DE$. $H$ là trực tâm của tam giác $ABC$. Chứng minh rằng đường thẳng qua $H$ vuông góc với $AM$ luôn đi qua một điểm cố định khi $A$ di chuyển trên $(O)$.

$(EDBC)=-1$ mà $M$ là trung điểm $ED$ suy ra $MD^2=MB.MC$ suy ra $MA$ là tiếp tuyến tại $M$ của $(O)$. Gọi $(d)$ là đường thẳng qua $H$ vuông góc $MA$ suy ra $(d)$ song song $AO$. Gọi $O'$ đối xứng $O$ qua $BC$ suy ra $HO'$ song song $AO$ suy ra $(d)$ qua $O'$




#697531 Chứng minh nếu $G$ là một đồ thị không liên thông thì đồ thị bù của...

Đã gửi bởi CaptainCuong on 30-11-2017 - 22:38 trong Tổ hợp và rời rạc

Chứng minh nếu $G$ là một đồ thị không liên thông thì đồ thị bù của $G$ là một đồ thị liên thông.

 




#678643 Topic ôn thi hình học vào cấp 3 chuyên

Đã gửi bởi CaptainCuong on 26-04-2017 - 08:37 trong Hình học

$\boxed{\text{Bài Toán 61}}$ [Sưu tầm] Từ $A$ nằm ngoài đường tròn $(O;R)$ vẽ $2$ tiếp tuyến $AB,AC$ với đường tròn $(O)$ ($B,C$ là các tiếp điểm). Gọi $P,Q$ lần lượt là trung điểm của $AB,AC$. Từ điểm $M$ bất kỳ thuộc cạnh $PQ$ kẻ tiếp tuyến $MD$ của đường tròn. Chứng minh rằng: $MA=MD$

$PB^2=PA^2; QA^2=QC^2 \rightarrow PQ$ là trục đẳng phương của $(O;R)$ và $(A;0)$ vậy. Vậy phương tích từ D đến $(O;R)$ và $(A;0)$ bằng nhau $\rightarrow MD^2=MA^2$ suy ra đpcm 




#671768 $p+p^2+p^3+p^4=q!$

Đã gửi bởi CaptainCuong on 15-02-2017 - 23:14 trong Số học

Tìm các số nguyên tố $p,q$ sao cho: $p+p^2+p^3+p^4=q!$




#664449 $a+b+c=1$ tim GTLN $ P=(a-b)(b-c)(c-a)$

Đã gửi bởi CaptainCuong on 12-12-2016 - 11:36 trong Bất đẳng thức và cực trị

Lời giải :

Xét $P^2=(a-b)^2(b-c)^2(c-a)^2.$
Không mất tính tổng quát. Giả sử $a \geq b \geq c.$
Ta suy ra các đánh giá sau:
$(b-c)^2 \leq b^2, (c-a)^2 \leq a^2$
Suy ra $P^2 \leq a^2b^2(a-b)^2$
Áp dụng bđt Cauchy ta có :
$4P^2=2ab.2ab.(a-b)^2 \leq \frac{[2ab+2ab+(a-b)^2]^3}{27}=\frac{(a+b)^6}{27} \leq \frac{(a+b+c)^6}{27}=\frac{1}{27}$
$\Rightarrow P^2 \leq \frac{1}{27.4}=\frac{1}{108}$
$\Rightarrow P \leq \frac{1}{6\sqrt{3}}$
Đẳng thức xảy ra khi $(a,b,c)=(\frac{3+\sqrt{3}}{6},\frac{3-\sqrt{3}}{6},0)$

$ab$ chưa chắc dương nên ko thể cauchy $b-c<b<0$ thì sao bạn




#664446 $a+b+c=1$ tim GTLN $ P=(a-b)(b-c)(c-a)$

Đã gửi bởi CaptainCuong on 12-12-2016 - 11:24 trong Bất đẳng thức và cực trị

Tương tự câu bất đề sau http://diendantoanho...ptnk-2016-2017/




#664339 GPT: $x^5+10x^3+20x-18=0$

Đã gửi bởi CaptainCuong on 10-12-2016 - 23:57 trong Phương trình - hệ phương trình - bất phương trình

 

Ta sẽ đăt: $x=\sqrt{2}(t-\dfrac{1}{t})$

 

Khi đó thay vào pt ta có:

 

$2\sqrt{2}(t-\dfrac{1}{t})^5+20\sqrt{2}(t-\dfrac{1}{t})^3+40\sqrt{2}(t-\dfrac{1}{t})-18=0$

 

$\iff 4\sqrt{2}(t^5-\dfrac{1}{t^5})=18$

 

$\iff 4\sqrt{2}t^{10}-18t^5-4\sqrt{2}=0$

 

Đến đây ta được phương trình bậc 2, giải ra $t$

 

Từ đó ta có nghiệm duy nhất của pt:

 

$x=\sqrt{2}(\sqrt[5]{\dfrac{9+\sqrt{113}}{4\sqrt{2}}}-\sqrt[5]{\dfrac{4\sqrt{2}}{9+\sqrt{113}}})$

 
 

 

Sao anh lại đặt được ẩn phụ $x=\sqrt{2}(t-\dfrac{1}{t})$




#661200 Chứng minh : MA = MD

Đã gửi bởi CaptainCuong on 08-11-2016 - 21:11 trong Hình học

Gọi giao điểm đường trung trực của $AD$ với tiếp tuyến tại $D$ ($D$ di động trên $O$) là $M'$ 

Ta có $M'D^2=M'A^2\Rightarrow M'\in$ trục đẳng phương của $(A;0)$ và $(O)$

Lại có $PA^2=PB^2\Rightarrow P\in$ trục đẳng phương của $(A;0)$ và $(O)$

$QA^2=QC^2\Rightarrow Q\in$ trục đẳng phương của $(A;0)$ và $(O)$

$\Rightarrow$ $M',P,Q$ thẳng hàng $\Rightarrow M\equiv M'$




#661047 CMR: $\sum \frac{(2a+b+c)2}{2a^2+(b+c)^2}...

Đã gửi bởi CaptainCuong on 07-11-2016 - 21:19 trong Bất đẳng thức - Cực trị

Một cách của thầy Cẩn

Để ý rằng $3-\frac{(2a+b+c)^2}{2a^2+(b+c)^2}=\frac{2(a+b-c)^2}{2a^2+(a+b)^2}$ nên bất đẳng thức cần C/m tương đương$\sum \frac{2(a+b-c)^2}{2a^2+(a+b)^2}\geq 1\Leftrightarrow \sum \frac{2(a+b-c)^2}{2a^2+2(b^2+c^2)}\geq 1\Leftrightarrow \sum \frac{(a+b-c)^2}{a^2+b^2+c^2}\geq 1$

$\Leftrightarrow (b+c-a)^2+(c+a-b)^2+(a+b-c)^2\geq a^2+b^2+c^2$

Bất đẳng thức này đúng do bất đẳng thức sau $\frac{(b+c-a)^2+(c+a-b)^2}{2}\geq c^2$




#660759 Chứng minh $MA$ vuông góc với $BC$

Đã gửi bởi CaptainCuong on 06-11-2016 - 07:59 trong Hình học

$\underset{AM}{\rightarrow}.\underset{BC}{\rightarrow}=0\Leftrightarrow (\underset{AE}{\rightarrow}+\underset{AD}{\rightarrow})(\underset{AC}{\rightarrow}-\underset{AB}{\rightarrow})=0\Leftrightarrow \underset{AE}{\rightarrow}\underset{AC}{\rightarrow}-\underset{AE}{\rightarrow}\underset{AB}{\rightarrow}+\underset{AD}{\rightarrow}\underset{AC}{\rightarrow}-\underset{AD}{\rightarrow}\underset{AB}{\rightarrow}=0\Leftrightarrow\underset{AE}{\rightarrow}\underset{AB}{\rightarrow}=\underset{AD}{\rightarrow}\underset{AC}{\rightarrow}$

$\Leftrightarrow AB.AE.cos\widehat{BAE}=AC.AD.cos\widehat{CAD}(TRUE)$

Hình gửi kèm

  • geogebra-export.png



#657324 $\frac{3(a+b+c)}{2(ab+bc+ca)}\geq \su...

Đã gửi bởi CaptainCuong on 09-10-2016 - 20:55 trong Bất đẳng thức - Cực trị

Bài toán 1: Cho $a,b,c>0$ thỏa mãn $a+b+c=6$. Cm:

 

\[\sqrt{\frac{{{\left( a+b \right)}^{3}}}{a+1}}+\sqrt{\frac{{{\left( b+c \right)}^{3}}}{b+2}}+\sqrt{\frac{{{\left( c+a \right)}^{3}}}{c+3}}\ge 12\]

$VT= \frac{(a+b)^2}{\sqrt{(a+1)(a+b)}}+\frac{(b+c)^2}{\sqrt{(b+2)(b+c)}}+\frac{(c+a)^2}{\sqrt{(c+3)(a+c)}}\geq \frac{144}{\sqrt{(a+1)(a+b)}+\sqrt{(b+2)(b+c)}+\sqrt{(c+3)(a+c)}}$

Có $\sqrt{(a+1)(a+b)}+\sqrt{(b+2)(b+c)}+\sqrt{(c+3)(a+c)}\leq\frac{a+1+a+b}{2}+\frac{b+2+b+c}{2}+\frac{c+3+a+c}{2}=12$

$\Rightarrow VT\geq \frac{144}{12}=12$




#657305 $\frac{3(a+b+c)}{2(ab+bc+ca)}\geq \su...

Đã gửi bởi CaptainCuong on 09-10-2016 - 20:05 trong Bất đẳng thức - Cực trị

$\sqrt{a^2b+b^2c}\leq \sqrt{\frac{a+a+b}{3}+\frac{b+b+c}{3}}=\sqrt{\frac{3b+2a+c}{3}}$

Đặt $\frac{3b+2a+c}{3}=x; \frac{3c+2b+a}{3}=y;\frac{3a+2c+b}{3}=z\rightarrow x+y+z=6$

$\Rightarrow \sum \sqrt{x}\leq \sqrt{3\sum x}=3\sqrt{2}$




#655871 $P=(a^2+1)^2+(b^2+1)^2+(c^2+1)^2+6\sqrt{6}abc$

Đã gửi bởi CaptainCuong on 28-09-2016 - 18:33 trong Bất đẳng thức - Cực trị

Cho $a,b,c$ thực thỏa mãn $a+b+c=0$

Tìm $GTNN$ của $P=(a^2+1)^2+(b^2+1)^2+(c^2+1)^2+6\sqrt{6}abc$




#654817 Hỏi đáp về GeoGebra

Đã gửi bởi CaptainCuong on 19-09-2016 - 21:44 trong Vẽ hình trên diễn đàn

Vẽ 2 góc = nhau ntn thầy ạ VD: Cho tam giác $ABC$ có trung tuyến $AM$. Lấy $P$ trong tam giác sao cho $\widehat{MAB}=\widehat{CAP}$




#653680 Tìm min: $F=3x^2+3y^2+z^2$

Đã gửi bởi CaptainCuong on 11-09-2016 - 10:09 trong Bất đẳng thức và cực trị

Ta có:

$x^2+y^2\geq 2xy$

$2y^2+\frac{1}{2}z^2\geq2yz$

$2x^2+\frac{1}{2}z^2\geq2zx$

$\Rightarrow 3x^2+3y^2+z^2\geq 2(xy+yz+zx)=10$




#651900 Topic về phương trình và hệ phương trình

Đã gửi bởi CaptainCuong on 29-08-2016 - 22:05 trong Phương trình - hệ phương trình - bất phương trình

Lời giải.

 

 

Lời giải.

Các cách thực sự rất hay. Chị có thể cho em ý tưởng cách đặt ẩn và biến đổi "ảo diệu" hoặc tài liệu về PP trên dc ko ạ?




#650296 $\frac{a}{b+c}+\frac{b}{c+a...

Đã gửi bởi CaptainCuong on 18-08-2016 - 23:37 trong Bất đẳng thức và cực trị

Cho $a,b,c>0$. Chứng minh $\frac{a}{b+c}+\frac{b}{c+a}+\frac{c}{a+b}+\sqrt{\frac{ab+bc+ca}{a^2+b^2+c^2}}\geq \frac{5}{2}$

 

 

P/s: Khuyến khích tách thành tổng bình phương




#648318 Topic về phương trình và hệ phương trình

Đã gửi bởi CaptainCuong on 06-08-2016 - 23:02 trong Phương trình - hệ phương trình - bất phương trình

Bài 471: Giải hệ phương trình:

$$\left\{\begin{matrix}a^2-b^2-2a+2b+3=0(1) \\ a^2-2ab+2b+7=0(2) \end{matrix}\right.$$

$8PT(1)-3PT(2)=0\Leftrightarrow (5a-4b-1)(a+2b-3)=0$




#647131 Cho x, y, z>0 : CMR: $\sum \frac{x^{3}...

Đã gửi bởi CaptainCuong on 30-07-2016 - 01:10 trong Bất đẳng thức và cực trị

À với cách đặt như vậy thì $abc=1$ nên em liên tưởng tới BĐT $vasc: \;\;\;\;\;\;\;\ \sum \frac{1}{a^{2k}+a^{k}+1}\geq 1.$

Ta sẽ chứng minh: $\frac{1}{(1+a)^3}\geq \frac{{\color{Red} 3}}{{\color{Red} 8}(a^{2k}+a^k+1)}\Leftrightarrow 8a^{2k}+8a^k+5\geq 3a^3+9a^2+9a$

Để có được như vậy thì đẳng thức phải xảy ra, tức là $8a^{2k}+8a^k+5= 3a^3+9a^2+9a$

Đạo hàm cả $2$ vế thì được $16k.a^{2k-1}+8k.a^{k-1}=9a^2+18a+9$

Mà dấu $"="$ xảy ra khi $x=y=z$ nên $a=1,$ từ đó tính được $k=\frac{3}{2}.$

Việc cuối cùng chỉ là đi chứng minh BĐT: $\frac{1}{(1+a)^3}\geq \frac{3}{8(a^3+\sqrt{a^3}+1)}$ là đúng.

---------------------------------------

Em cũng không biết tại sao anh lại nghĩ đến chứng minh BĐT: $(1+a)^3 \leq (1+abc)(1+\frac{a}{b})(1+\frac{a}{c})$

Vậy còn hai số 3 và 8 này em chọn như thế nào?




#639942 Đề thi tuyển sinh lớp 10 chuyên toán TPHCM 2016-2017

Đã gửi bởi CaptainCuong on 12-06-2016 - 23:19 trong Tài liệu - Đề thi

$\widehat{C_1B_1H}=\widehat{HCA_1}\Rightarrow \Delta BB_1L\sim \Delta HCA_1\Rightarrow \frac{LB_1}{A_1C}=\frac{BB_1}{HC}\Rightarrow LB_1=\frac{BB_1.A_1C}{HC}$

Cmtt, ta có:

$A_1K=\frac{BB_1.AK}{AB_1}(do \Delta AKA_1\sim \Delta ABB_1)$

$\Rightarrow \frac{LB_1}{A_1K}=\frac{A_1C.AB_1}{HC.AK}$

mà $\Delta AB_1K\sim \Delta CHA_1\Rightarrow \frac{A_1C}{AK}=\frac{HC}{AB_1}\Rightarrow \frac{A_1C.AB_1}{HC.AK}=1$

$\Rightarrow \frac{LB_1}{A_1K}=1\Rightarrow LB_1=A_1K$

Hình gửi kèm

  • 2.png



#637690 Đề thi tuyển sinh lớp 10 chuyên toán PTNK 2016-2017

Đã gửi bởi CaptainCuong on 02-06-2016 - 21:28 trong Tài liệu - Đề thi

Cách này với cách mình post hình như giống nhau

:luoi: xin lỗi bạn mình không để ý




#637681 Đề thi tuyển sinh lớp 10 chuyên toán PTNK 2016-2017

Đã gửi bởi CaptainCuong on 02-06-2016 - 20:58 trong Tài liệu - Đề thi

Còn cách đơn giản hơn:

Đặt $a=x+y$ và $b=x-y$ $(a,b$ chẵn$)$

Từ giả thiết$=>\frac{a^2+10}{a^2-b^2}=\frac{k+2}{4}$

Đặt $d=(a^2+10,a^2-b^2)$

Dễ thấy $d=2$ hoặc $d=10$

Mặt khác do $4\mid k$:

Xét $k=4=>\frac{a^2+10}{a^2-b^2}=\frac{3}{2}$

$=>PT$ vô nghiệm

Xét $k=8=>\frac{a^2+10}{a^2-b^2}=\frac{5}{2}$

$=>PT$ vô nghiệm

Suy ra $k=4$ hoặc $k=8$ không thỏa mãn nên $k\geqslant 12$

Mình còn cách gọn hơn tí :)

C/m được số chính phương chia 3 dư 0 hoặc 1

+Xét 2 số $x,y$, có 1 số chia hết cho 3, 1 số không chia hết cho 3. Không mất tính tổng quát giả sử $x$ chia hết cho 3, $y$ không chia hết cho 3 $\Rightarrow x^2+y^2+10\equiv 2(mod 3)$ mà xy chia hết cho 3 $\Rightarrow$ loại.

+ Xét 2 số $x,y$ đều chia hết cho 3 $\Rightarrow x^2,y^2\vdots 9\Rightarrow x^2+y^2+10\equiv 1(mod 9)$ mà $xy$ chia hết cho 9  $\Rightarrow$ loại.

Vậy $x,y$ không chia hết cho 3 $\Rightarrow x^2,y^2\equiv 1(mod 3)\Rightarrow x^2+y^2+10\equiv 0(mod 3)$ mà $xy$ không chia hết cho 3 $\Rightarrow$ $k\vdots 3$

mà $k\vdots 4$

$(4;3)=1$

$\Rightarrow$ $k\vdots 12$ mà $k\neq 0\Rightarrow k\geq 12$